If Fluffy is displayed next to Jaguar, the display CANNOT include

Ana Lorenza on March 20, 2020

September 2019 LSAT Question 4

Hi! I don't understand why Wags is the correct answer here. The answer doesn't give an explanation. Could someone please explain why? Thanks so much!

Reply
Create a free account to read and take part in forum discussions.

Already have an account? log in

Ravi on March 20, 2020

@Lorenza,

Happy to help. Let's take a look and start with the general game setup first.

We have 4 kittens: F, G, H, and J

and 4 puppies: R, S, T, and W

Here are the rules:

1) pens 1 and 5 have to have kittens
2) T can't go next to a kitten
3) G or H, but not both, is in
4) if W is in, then G has to be in 2

Thinking about the rules, let's look at rules 1 and 2. If T is in, he
can't touch a kitten, so that means he can't be in 2 or 4. Thus, he
has to go in 3 if he's in

_ _ T _ _

and in this scenario, 2 and 4 would also have to be dogs

The other scenario is way more open-ended. T would be out, along with
G/H and someone else. H/G would be in, along with to kittens for sure
in 1 and 5

_ _ _ _ _

Question 4 tells us that F is next to J. If F is next to J, then that
means that there is no way that T is in since either 2, 3, or 4 have
to be kittens and if T is in, then, 2, 3, and 4 are all puppies.
Therefore, we're looking at scenario 2.

Looking further at this scenario, we see that G/H is already out and
kittens are already in 1 and 5. This means that in order for 2 kittens
to touch, that could only happen if they touched with 1 and 2 or 4 and
5, so there are only two possibilities within this scenario for F and
J to touch.

FJ _ _ _
_ _ _ FJ

In the top scenario, J or F is in 2, so G isn't in 2, which means W is out.

In the bottom scenario, if F and J are in 4 and 5, then that means
that if G is in, the only spot it could go is in 1 since H would have
to be out of the display. Therefore, G isn't in 2 in this scenario
either. From this, we know that W is out, since this is failing the
necessary condition of rule 4. As a result, no matter what, W will be
out, so (E) is the correct answer choice.

Does this make sense? Let us know if you have any other questions!